Download as docx, pdf, or txt
Download as docx, pdf, or txt
You are on page 1of 69

Chapter 17 - Allocation of Support Activity Costs and Joint Costs

Chapter 17
Allocation of Support Activity Costs and Joint Costs
Answer Key
 

True / False Questions


 

1. The direct method ignores the fact that some service departments provide service to other
service departments. 
TRUE

AACSB: Reflective Thinking


AICPA BB: Industry
AICPA FN: Decision Making
Blooms: Remember
Difficulty: 1 Easy
Learning Objective: 17-01
Feedback True: Correct! It is true that the direct method ignores the fact that some service departments provide service to other service
departments.
Feedback False: It is true that the direct method ignores the fact that some service departments provide service to other service departments.

2. The step-down method of service department cost allocation completely ignores the
provision of services by one service department to another service department.
FALSE

AACSB: Reflective Thinking


AICPA BB: Industry
AICPA FN: Decision Making
Blooms: Remember
Difficulty: 1 Easy
Learning Objective: 17-01
Feedback True: This is a shortcoming of the direct method, which is partially overcome by the step-down method of service department cost
allocation.
Feedback False: Correct! This is a shortcoming of the direct method, which is partially overcome by the step-down method of service
department cost allocation.

17-1
Chapter 17 - Allocation of Support Activity Costs and Joint Costs

3. City Hospital has two service departments (Patient Records and Accounting) and two
"production" departments (Internal Medicine and Surgery). It uses the reciprocal-services
method of cost allocation and should allocate Internal Medicine cost to Surgery. 
FALSE

 
AACSB: Reflective Thinking
AICPA BB: Critical Thinking
AICPA FN: Research
Blooms: Understand
Difficulty: 2 Medium
Learning Objective: 17-01
Feedback True: The reciprocal-services method of cost allocation would not be used to allocate Internal Medicine cost to Surgery. 
Feedback False: Correct! The reciprocal-services method of cost allocation would not be used to allocate Internal Medicine cost to
Surgery. 

4. When allocating service department costs, companies should use actual costs rather than
budgeted costs, and separate rates for variable and fixed costs. 
FALSE

 
AACSB: Reflective Thinking
AICPA BB: Critical Thinking
AICPA FN: Research
Blooms: Remember
Difficulty: 1 Easy
Learning Objective: 17-02
Feedback True: Companies should not use actual costs and separate rates from variable and fixed costs when allocating service department
costs.
Feedback False: Correct! Companies should not use actual costs and separate rates from variable and fixed costs when allocating service
department costs.

5. Dual cost allocation works with either the direct method or the step-down method of
allocation.
TRUE

AACSB: Reflective Thinking


AICPA BB: Critical Thinking
AICPA FN: Research
Blooms: Remember
Difficulty: 1 Easy
Learning Objective: 17-02
Feedback True: Correct! Dual cost allocations works with either the direct method or the step-down method of allocation.
Feedback False: Dual cost allocations works with either the direct method or the step-down method of allocation.

17-2
Chapter 17 - Allocation of Support Activity Costs and Joint Costs

6. A company that uses activity-based costing would likely allocate costs from activity-cost
pools to products and services. 
TRUE

AACSB: Reflective Thinking


AICPA BB: Critical Thinking
AICPA FN: Research
Blooms: Remember
Difficulty: 1 Easy
Learning Objective: 17-03
Feedback True: Correct! It is true that a company that uses activity-based costing would likely allocate costs from activity-cost pools to
products and services. 
Feedback False: It is true that a company that uses activity-based costing would likely allocate costs from activity-cost pools to products
and services.
 

7. The breakdown of costs by departments is much finer than a breakdown by activities in an


ABC system.
FALSE

 
AACSB: Reflective Thinking
AICPA BB: Critical Thinking
AICPA FN: Research
Blooms: Understand
Difficulty: 2 Medium
Learning Objective: 17-03 
Feedback True: The breakdown of costs by activities using ABC is much finer than a breakdown of costs by department.
Feedback False: Correct! The breakdown of costs by activities using ABC is much finer than a breakdown of costs by department.

8. The Gross Margin at Split-Off method should be selected if a company terminates all
processing at the split-off point and desires to use a cost-allocation approach that considers
the "revenue-producing ability" of each product. 
FALSE

 
AACSB: Reflective Thinking
AICPA BB: Critical Thinking
AICPA FN: Research
Blooms: Understand
Difficulty: 2 Medium
Learning Objective: 17-04 
Feedback True: The Gross Margin at Split-Off method is not selected if a company terminates all processing at the split-off point and desires
to use a cost-allocation approach to consider the revenue producing ability of each product.
Feedback False: Correct! The Gross Margin at Split-Off method is not selected if a company terminates all processing at the split-off point
and desires to use a cost-allocation approach to consider the revenue producing ability of each product.

17-3
Chapter 17 - Allocation of Support Activity Costs and Joint Costs

9. In the net-realizable-value method, the joint cost is allocated to the joint products in
proportion to the joint products’ net realizable values.
TRUE

 
AACSB: Reflective Thinking
AICPA BB: Critical Thinking
AICPA FN: Research
Blooms: Understand
Difficulty: 2 Medium
Learning Objective: 17-04 
Feedback True: Correct! Joint cost is allocated to joint products in proportion to the joint products’ net realizable values using the NRV
method.
Feedback False: Joint cost is allocated to joint products in proportion to the joint products’ net realizable values using the NRV method.

10. The net-realizable-value method is the least preferred cost allocation method since it does
not consider the economic characteristics of the joint products.
FALSE

 
AACSB: Reflective Thinking
AICPA BB: Critical Thinking
AICPA FN: Research
Blooms: Understand
Difficulty: 2 Medium
Learning Objective: 17-05 
Feedback True: The physical-units approach is the least preferred cost allocation method since it does not consider the economic
characteristics of the joint products.
Feedback False: Correct! The physical-units approach is the least preferred cost allocation method since it does not consider the economic
characteristics of the joint products.

11. A joint product with minimal value relative to the other joint products is known as a by-
product.
TRUE

 
AACSB: Reflective Thinking
AICPA BB: Critical Thinking
AICPA FN: Research
Blooms: Understand
Difficulty: 2 Medium
Learning Objective: 17-05 
Feedback True: Correct! This is the definition of a by-product.
Feedback False: This is the definition of a by-product.

12. The reciprocal-services method is less accurate than the direct and step-down methods.
FALSE
AACSB: Reflective Thinking
AICPA BB: Critical Thinking
AICPA FN: Research
Blooms: Understand
Difficulty: 2 Medium
Learning Objective: 17-06 
Feedback True: The reciprocal-services method is more accurate than the direct and step-down methods.
Feedback False: Correct! The reciprocal-services method is more accurate than the direct and step-down methods.

17-4
Chapter 17 - Allocation of Support Activity Costs and Joint Costs

13. The reciprocal-services method cannot be combined with the dual-allocation approach.
FALSE
AACSB: Reflective Thinking
AICPA BB: Critical Thinking
AICPA FN: Research
Blooms: Understand
Difficulty: 2 Medium
Learning Objective: 17-06 
Feedback True: The reciprocal-services method can be made even more accurate than the direct and step-down methods when combined
with the dual-allocation approach.
Feedback False: Correct! The reciprocal-services method can be made even more accurate than the direct and step-down methods when
combined with the dual-allocation approach.

Multiple Choice Questions


 

14. Which of the following would be considered a service department for an airline? 


A. Maintenance.
B. Information Systems.
C. Purchasing.
D. Flight Catering.
E. All of the answers are correct.

 
AACSB: Reflective Thinking
AICPA BB: Industry
AICPA FN: Decision Making
Blooms: Understand
Difficulty: 2 Medium
Learning Objective: 17-01
Feedback A: This would be a service department for an airline, but there is a better answer choice.
Feedback B: This would be a service department for an airline, but there is a better answer choice.
Feedback C: This would be a service department for an airline, but there is a better answer choice.
Feedback D: This would be a service department for an airline, but there is a better answer choice.
Feedback E: Correct! All of these would be service departments for an airline.

17-5
Chapter 17 - Allocation of Support Activity Costs and Joint Costs

15. Which of the following would not be considered a service department in a hospital? 


A. Security.
B. Cardiac Care.
C. Patient Records.
D. Accounting.
E. Human Resources.

 
AACSB: Reflective Thinking
AICPA BB: Industry
AICPA FN: Decision Making
Blooms: Remember
Difficulty: 1 Easy
Learning Objective: 17-01
Feedback A: This would be a service department for a hospital.
Feedback B: Correct! This would not be a service department for a hospital.
Feedback C: This would be a service department for a hospital.
Feedback D: This would be a service department for a hospital.
Feedback E: This would be a service department for a hospital.

16. Consider the following statements about service department costs:


I. The costs of the Human Resources Department in a manufacturing organization must be
allocated to production departments in order to achieve a correct costing of inventory.
II. The allocation of service department costs requires that an organization select both an
allocation base and an allocation method.
III. Service department cost allocations are more relevant for firms involved in service
industries (e.g., repair, health care) than for those involved with manufacturing.
Which of the above statements is (are) correct? 
A. I only.
B. II only.
C. I and II.
D. II and III.
E. I, II, and III.

AACSB: Reflective Thinking


AICPA BB: Industry
AICPA FN: Decision Making
Blooms: Remember
Difficulty: 1 Easy
Learning Objective: 17-01
Feedback A: While this statement is correct, there is a better answer choice.
Feedback B: While this statement is correct, there is a better answer choice.
Feedback C: Correct! Both statements I and II are correct.
Feedback D: One of these statements is incorrect.
Feedback E: One of these statements is incorrect.

17-6
Chapter 17 - Allocation of Support Activity Costs and Joint Costs

17. Which of the following methods ignores the fact that some service departments provide
service to other service departments? 
A. Direct method.
B. Indirect method.
C. Step-down method.
D. Reciprocal method.
E. Dual-cost allocation method.

 
AACSB: Reflective Thinking
AICPA BB: Industry
AICPA FN: Decision Making
Blooms: Remember
Difficulty: 1 Easy
Learning Objective: 17-01
Feedback A: Correct! The direct method ignores service departments providing service to other service departments.
Feedback B: This method does not ignore this point.
Feedback C: This method does not ignore this point.
Feedback D: This method does not ignore this point.
Feedback E: This method does not ignore this point.
 

18. Which of the following methods fully recognizes the fact that some service departments
provide service to other service departments? 
A. Direct method.
B. Indirect method.
C. Step-down method.
D. Reciprocal method.
E. Dual-cost allocation method.

 
AACSB: Reflective Thinking
AICPA BB: Industry
AICPA FN: Decision Making
Blooms: Remember
Difficulty: 1 Easy
Learning Objective: 17-01
Feedback A: This method does not fully recognize this fact.
Feedback B: This method does not fully recognize this fact.
Feedback C: This method does not fully recognize this fact.
Feedback D: Correct! The reciprocal method completely recognizes the fact that some service departments provide service to other service
departments.
Feedback E: This method does not fully recognize this fact.

17-7
Chapter 17 - Allocation of Support Activity Costs and Joint Costs

19. Which of the following methods recognizes the fact that fixed and variable service
department costs should be allocated separately? 
A. Direct method.
B. Indirect method.
C. Step-down method.
D. Reciprocal method.
E. Dual-cost allocation method.

 
AACSB: Reflective Thinking
AICPA BB: Industry
AICPA FN: Decision Making
Blooms: Remember
Difficulty: 1 Easy
Learning Objective: 17-01
Feedback A: This method does not recognize fixed and variable service department cost separation when allocating costs.
Feedback B: This method does not recognize fixed and variable service department cost separation when allocating costs.
Feedback C: This method does not recognize fixed and variable service department cost separation when allocating costs.
Feedback D: This method does not recognize fixed and variable service department cost separation when allocating costs.
Feedback E: Correct! The dual-cost allocation method recognizes the fact that fixed and variable service department costs should be
allocated separately.

20. Consider the following statements about the direct method of service department cost
allocation:
I. Under the direct method, all service department costs are eventually allocated to production
departments.
II. The order in which service department costs are allocated to production departments is
important.
III. Once a service department's costs have been allocated, no costs are re-circulated back to
that department.

Which of the above statements is (are) correct? 


A. I only.
B. II only.
C. I and II.
D. I and III.
E. I, II, and III.

 
AACSB: Reflective Thinking
AICPA BB: Critical Thinking
AICPA FN: Decision Making
Blooms: Understand
Difficulty: 2 Medium
Learning Objective: 17-01
Feedback A: While this statement is correct, there is a better answer choice.
Feedback B: This statement is incorrect.
Feedback C: One of these statements is incorrect.
Feedback D: Correct! Both statements I and III are correct.
Feedback E: One of these statements is incorrect.

17-8
Chapter 17 - Allocation of Support Activity Costs and Joint Costs

21. The Buckaneer Clinic has two service departments (Human Resources and Information
Resources) and two "production" departments (In-patient Treatment and Out-patient
Treatment). The service departments service the "production" departments as well as each
other, and studies have shown that Information Resources provides the greater amount of
service. Which of the following allocations would occur if Buckaneer uses the direct method
of cost allocation? 
A. Information Resources cost would be allocated to In-patient Treatment.
B. Information Resources cost would be allocated to Human Resources.
C. Human Resources cost would be allocated to Information Resources.
D. In-patient Treatment cost would be allocated to Out-patient Treatment.
E. Out-patient Treatment cost would be allocated to Information Resources.

 
AACSB: Reflective Thinking
AICPA BB: Critical Thinking
AICPA FN: Decision Making
Blooms: Understand
Difficulty: 2 Medium
Learning Objective: 17-01
Feedback A: Correct! Information Resources cost would be allocated to In-patient Treatment.
Feedback B: This would not occur using the direct method.
Feedback C: This would not occur using the direct method.
Feedback D: This would not occur using the direct method.
Feedback E: This would not occur using the direct method.

22. Tennison Corporation has two service departments (Maintenance and Human Resources)
and three production departments (Machining, Assembly, and Finishing). Maintenance is the
larger service department and Assembly is the largest production department. The two service
departments service each other as well as the three producing departments. On the basis of
this information, which of the following cost allocations would not occur under the direct
method? 
A. Machining cost would be allocated to Assembly.
B. Maintenance cost would be allocated to Finishing.
C. Maintenance cost would be allocated to Human Resources.
D. Human Resources cost would be allocated to Finishing.
E. Both machining cost would be allocated to Assembly and maintenance cost would be
allocated to Human Resources.

AACSB: Reflective Thinking


AICPA BB: Critical Thinking
AICPA FN: Decision Making
Blooms: Understand
Difficulty: 2 Medium
Learning Objective: 17-01
Feedback A: While this could not occur, there is a better answer choice.
Feedback B: This could occur using the direct method.
Feedback C: While this could not occur, there is a better answer choice.
Feedback D: This could occur using the direct method.
Feedback E: Correct! Both machining cost allocated to Assembly and maintenance cost allocated to Human Resources could not occur
using the direct method. 

17-9
Chapter 17 - Allocation of Support Activity Costs and Joint Costs

23. Which of the following methods recognizes some (but not all) of the services that occur
between service departments? 
A. Direct method.
B. Step-down method.
C. Indirect method.
D. Reciprocal method.
E. Dual-cost allocation method.

AACSB: Reflective Thinking


AICPA BB: Critical Thinking
AICPA FN: Decision Making
Blooms: Remember
Difficulty: 1 Easy
Learning Objective: 17-01
Feedback A: This method does not satisfy the criterion listed.
Feedback B: Correct! The step-down method recognizes some but not all of the services that occur between service departments.
Feedback C: This method does not satisfy the criterion listed.
Feedback D: This method does not satisfy the criterion listed.
Feedback E: This method does not satisfy the criterion listed.

24. When the step-down method is used, the service department whose costs are allocated
first is often the department that: 
A. obtains the highest yield.
B. has the lowest cost.
C. is the newest.
D. serves the greatest number of other service departments.
E. serves the fewest other service departments.

 
AACSB: Reflective Thinking
AICPA BB: Critical Thinking
AICPA FN: Decision Making
Blooms: Remember
Difficulty: 1 Easy
Learning Objective: 17-01
Feedback A: This statement is incorrect.
Feedback B: This statement is incorrect.
Feedback C: This statement is incorrect.
Feedback D: Correct! When the step-down method is used, the service department whose costs are allocated first is often the department
that serves the greatest number of other service departments.
Feedback E: This statement is incorrect.
 

17-10
Chapter 17 - Allocation of Support Activity Costs and Joint Costs

25. Consider the following statements about the step-down method of service department cost
allocation:
I. Under the step-down method, all service department costs are eventually allocated to
production departments.
II. The order in which service department costs are allocated is important.
III. After a service department's costs have been allocated to other departments, no costs are
re-circulated back to that service department.

Which of the above statements is (are) correct? 


A. I only.
B. II only.
C. I and II.
D. I and III.
E. I, II, and III.

 
AACSB: Reflective Thinking
AICPA BB: Critical Thinking
AICPA FN: Decision Making
Blooms: Remember
Difficulty: 1 Easy
Learning Objective: 17-01
Feedback A: While this statement is correct, there is a better answer choice.
Feedback B: While this statement is correct, there is a better answer choice.
Feedback C: While these statements are correct, there is a better answer choice.
Feedback D: While these statements are correct, there is a better answer choice.
Feedback E: Correct! All of these statements are correct.

26. Oxmoor Corporation has two service departments (Maintenance and Human Resources)
and three production departments (Machining, Assembly, and Finishing). The two service
departments service the production departments as well as each other, and studies have shown
that Maintenance provides the greater amount of service. On the basis of this information,
which of the following cost allocations would likely occur under the step-down method? 
A. Machining cost would be allocated to Assembly.
B. Maintenance cost would be allocated to Finishing.
C. Maintenance cost would be allocated to Human Resources.
D. Human Resources cost would be allocated to Maintenance.
E. Both maintenance cost would be allocated to Finishing and maintenance cost would be
allocated to Human Resources.

 
AACSB: Reflective Thinking
AICPA BB: Critical Thinking
AICPA FN: Decision Making
Blooms: Understand
Difficulty: 2 Medium
Learning Objective: 17-01
Feedback A: This would not occur.
Feedback B: While this could occur, there is a better answer choice listed.
Feedback C: While this could occur, there is a better answer choice listed.
Feedback D: This would not occur.
Feedback E: Correct! Both of these would likely occur using a step-down allocation method.

17-11
Chapter 17 - Allocation of Support Activity Costs and Joint Costs

27. The Covington Clinic has two service departments (Human Resources and Information
Systems) and two "production" departments (In-patient Treatment and Out-patient
Treatment). The service departments service the "production" departments as well as each
other, and studies have shown that Information Systems provides the greater amount of
service. Which of the following allocations would not occur if Covington uses the step-down
method of cost allocation? 
A. Information Systems cost would be allocated to Human Resources.
B. Human Resources cost would be allocated to Information Systems.
C. Human Resources cost would be allocated to In-patient Treatment.
D. In-patient Treatment cost would be allocated to Out-patient Treatment.
E. Both Human Resources cost would be allocated to Information Systems and In-patient
Treatment cost would be allocated to Out-patient Treatment.

AACSB: Reflective Thinking


AICPA BB: Critical Thinking
AICPA FN: Decision Making
Blooms: Understand
Difficulty: 2 Medium
Learning Objective: 17-01
Feedback A: This could possibly occur.
Feedback B: While this could not occur, there is a better answer choice listed.
Feedback C: This could possibly occur.
Feedback D: While this could not occur, there is a better answer choice listed.
Feedback E: Correct! Both of these likely would not occur using a step-down allocation method.
 

28. The Blue Sky Clinic has two service departments (S1 and S2) and two "production"
departments (P1 and P2). The service departments service the "production" departments as
well as each other, and studies have shown that S2 provides the greater amount of service.
Which of the following choices correctly denotes an allocation that would not occur under (1)
the direct method and (2) the step-down method of cost allocation?

Direct Method Step-Down Method


A S1’s cost would be allocated to P1. S1’s cost would be allocated to P1.
B. P1’s cost would be allocated to P2. P1’s cost would be allocated to P2.
C. S2’s cost would be allocated to S1. S2’s cost would be allocated to S1.
D. P1’s cost would be allocated to S1. P1’s cost would be allocated to S1.
E. None of the answers is correct.

 
AACSB: Reflective Thinking
AICPA BB: Critical Thinking
AICPA FN: Decision Making
Blooms: Understand
Difficulty: 2 Medium
Learning Objective: 17-01
Feedback A: This could occur.
Feedback B: This could occur.
Feedback C: This could occur.
Feedback D: This could occur.
Feedback E: Correct! All of these could occur under the direct and step-down methods, so the answer is none of these.

17-12
Chapter 17 - Allocation of Support Activity Costs and Joint Costs

29. Which of the following methods accounts for 100% of the services that occur between
service departments? 
A. Direct method.
B. Indirect method.
C. Reciprocal method.
D. Step-down method.
E. Dual-cost allocation method.

 
AACSB: Reflective Thinking
AICPA BB: Critical Thinking
AICPA FN: Decision Making
Blooms: Remember
Difficulty: 1 Easy
Learning Objective: 17-01
Learning Objective: 17-06
Feedback A: This is not the method where all services between service departments are accounted for.
Feedback B: This is not the method where all services between service departments are accounted for.
Feedback C: Correct! The reciprocal method accounts for 100% of the services that occur between service departments.
Feedback D: This is not the method where all services between service departments are accounted for.
Feedback E: This is not the method where all services between service departments are accounted for.
 

30. Naples Corporation has two service departments (Maintenance and Human Resources)
and three production departments (Machining, Assembly, and Finishing). The two service
departments service each other, and studies have shown that Maintenance provides the greater
amount of service. Given the various cost allocation methods, which of the following choices
correctly denotes whether Maintenance cost would be allocated to Human Resources?

Direct Step-Down Reciprocal


A Yes No Yes
B. Yes No No
C. Yes Yes Yes
D. No Yes No
E. No Yes Yes

AACSB: Reflective Thinking


AICPA BB: Critical Thinking
AICPA FN: Decision Making
Blooms: Understand
Difficulty: 2 Medium
Learning Objective: 17-01
Learning Objective: 17-06
Feedback A: This answer combination is incorrect.
Feedback B: This answer combination is incorrect.
Feedback C: This answer combination is incorrect.
Feedback D: This answer combination is incorrect.
Feedback E: Correct! Maintenance cost would not be allocated to HR using the direct method, but would be allocated using the step-down
and reciprocal methods.
 

17-13
Chapter 17 - Allocation of Support Activity Costs and Joint Costs

31. Which of the following methods would be of little use when allocating service department
costs to production departments? 
A. The direct method.
B. The reciprocal method.
C. The step-down method.
D. The net-realizable-value method.
E. The dual-cost allocation method.

 
AACSB: Reflective Thinking
AICPA BB: Critical Thinking
AICPA FN: Decision Making
Blooms: Understand
Difficulty: 2 Medium
Learning Objective: 17-01
Learning Objective: 17-02
Learning Objective: 17-04
Feedback A: This method would be useful when allocating service department costs to production departments.
Feedback B: This method would be useful when allocating service department costs to production departments.
Feedback C: This method would be useful when allocating service department costs to production departments.
Feedback D: Correct! This method would be of little use when allocating service department costs to production departments.
Feedback E: This method would be useful when allocating service department costs to production departments. 

17-14
Chapter 17 - Allocation of Support Activity Costs and Joint Costs

Use the following information to answer Questions 32 & 33.


Aldo Industries, Inc. has two service departments (Human Resources and Building
Maintenance) and two production departments (Machining and Assembly). The company
allocates Building Maintenance cost on the basis of square footage and believes that Building
Maintenance provides more service than Human Resources. The square footage occupied by
each department follows.

Human Resources 6,000


Building Maintenance 13,000
Machining 18,000
Assembly 26,000

32. Assuming use of the direct method, over how many square feet would the Building
Maintenance cost be allocated (i.e., spread)? 
A. 19,000.
B. 44,000.
C. 50,000.
D. 63,000.
E. More information is needed to judge.

AACSB: Analytic
AICPA BB: Critical Thinking
AICPA FN: Measurement
Blooms: Apply
Difficulty: 3 Hard
Learning Objective: 17-01
Feedback A: This amount is incorrect.
Feedback B: Correct! Machining + Assembly = 18,000 + 26,000 = 44,000 sq. ft.
Feedback C: This amount is incorrect.
Feedback D: This amount is incorrect.
Feedback E: There is sufficient information to make a numerical judgment.
 

33. Assuming use of the step-down method, over how many square feet would the Building
Maintenance cost be allocated (i.e., spread)? 
A. 19,000.
B. 44,000.
C. 50,000.
D. 63,000.
E. More information is needed to judge.

AACSB: Analytic
AICPA BB: Critical Thinking
AICPA FN: Measurement
Blooms: Apply
Difficulty: 3 Hard
Learning Objective: 17-01
Feedback A: This amount is incorrect.
Feedback B: This amount is incorrect.
Feedback C: Correct! Machining + Assembly + HR = 18,000 + 26,000 + 6,000 = 50,000 sq. ft.
Feedback D: This amount is incorrect.
Feedback E: There is sufficient information to make a numerical judgment.

17-15
Chapter 17 - Allocation of Support Activity Costs and Joint Costs

Use this information to answer questions 34-35.

Rave Reviews Company has two service departments (Cafeteria and Human Resources) and
two production departments (Machining and Assembly). The number of employees in each
department follows.
 
Cafeteria 40
Human Resources 60
Machining 200
Assembly 300

34. Rave Reviews uses the direct method of cost allocation and allocates cost on the basis of
employees. If Human Resources cost amounts to $1,800,000, how much of the department's
cost would be allocated to Machining? 
A. $600,000.
B. $720,000.
C. $900,000.
D. $1,200,000.
E. None of the answers is correct.

AACSB: Analytic
AICPA BB: Critical Thinking
AICPA FN: Measurement
Blooms: Apply
Difficulty: 3 Hard
Learning Objective: 17-01
Feedback A: This amount is incorrect.
Feedback B: Correct! 200 ÷ 500 = 40%; 40% x $1,800,000 = $720,000
Feedback C: This amount is incorrect.
Feedback D: This amount is incorrect.
Feedback E: This answer is wrong, because there is a correct amount listed.

35. Rave Reviews uses the direct method of cost allocation and allocates cost on the basis of
employees. If Human Resources cost amounts to $1,800,000, how much of the department's
cost would be allocated to Assembly? 
A. $900,000.
B. $720,000.
C. $1,080,000.
D. $1,200,000.
E. None of the answers is correct.

AACSB: Analytic
AICPA BB: Critical Thinking
AICPA FN: Measurement
Blooms: Apply
Difficulty: 3 Hard
Learning Objective: 17-01
Feedback A: This amount is incorrect.
Feedback B: This amount is incorrect.
Feedback C: Correct! 300 ÷ 500 = 60%; 60% x $1,800,000 = $1,080,000
Feedback D: This amount is incorrect.
Feedback E: This answer is wrong, because there is a correct amount listed.

17-16
Chapter 17 - Allocation of Support Activity Costs and Joint Costs

 
Use this information to answer questions 36-37.
Trek, Inc. has two service departments (Human Resources and Building Maintenance) and
two production departments (Machining and Assembly). The company allocates Building
Maintenance cost on the basis of square footage and believes that Building Maintenance
provides more service than Human Resources. The square footage occupied by each
department follows.
Human Resources 7,000
Building Maintenance 11,000
Machining 20,000
Assembly 28,000

 
36. Assuming use of the step-down method, over how many square feet would the Building
Maintenance cost be allocated (i.e., spread)? 
A. 18,000.
B. 48,000.
C. 55,000.
D. 66,000.
E. More information is needed to judge.

AACSB: Analytic
AICPA BB: Critical Thinking
AICPA FN: Measurement
Blooms: Apply
Difficulty: 3 Hard
Learning Objective: 17-01
Feedback A: This amount is incorrect.
Feedback B: This amount is incorrect.
Feedback C: Correct! HR + Machining + Assembly = 7,000 + 20,000 + 28,000 = 55,000 square feet.
Feedback D: This amount is incorrect.
Feedback E: There is sufficient information to make a numerical judgment.

37. Assuming use of the direct method, over how many square feet would the Building
Maintenance cost be allocated (i.e., spread)? 
A. 18,000.
B. 48,000.
C. 55,000.
D. 66,000.
E. More information is needed to judge.
AACSB: Analytic
AICPA BB: Critical Thinking
AICPA FN: Measurement
Blooms: Apply
Difficulty: 3 Hard
Learning Objective: 17-01
Feedback A: This amount is incorrect.
Feedback B: Correct! Machining + Assembly = 20,000 + 28,000 = 48,000 sq. ft.
Feedback C: This amount is incorrect.
Feedback D: This amount is incorrect.

17-17
Chapter 17 - Allocation of Support Activity Costs and Joint Costs

Feedback E: There is sufficient information to make a numerical judgment.

 
38. Sparkle Metallurgy, Inc. has two service departments (Human Resources and Building
Maintenance) and two production departments (Machining and Assembly). The company
allocates Building Maintenance cost on the basis of square footage and Human Resources cost
on the basis of employees. It believes that Building Maintenance provides more service than
Human Resources. The square footage and employees in each department follow.

Square
Footage Employees
Human Resources 4,000 10
Building Maintenance 10,000 15
Machining 15,000 40
Assembly 21,000 60

Assuming use of the step-down method, which of the following choices correctly denotes the
number of square feet and employees over which the Building Maintenance cost and Human
Resources cost would be allocated (i.e., spread)?

Building Maintenance Human


Resources
A. 36,000 100
B. 40,000 100
C. 46,000 110
D. 50,000 110
E. None of the answers is
correct.
 

AACSB: Analytic
AICPA BB: Critical Thinking
AICPA FN: Measurement
Blooms: Apply
Difficulty: 3 Hard
Learning Objective: 17-01
Feedback A: This amount combination is incorrect.
Feedback B: Correct! HR + Machining + Assembly = 4,000 + 15,000 + 21,000 = 40,000 sq. ft.; HR = Machining + Assembly = 40 + 60 =
100 employees.
Feedback C: This amount combination is incorrect.
Feedback D: This amount combination is incorrect.
Feedback E: This answer is wrong, because there is a correct combination listed.

17-18
Chapter 17 - Allocation of Support Activity Costs and Joint Costs

39. Visions, Inc. has two service departments (Human Resources and Building Maintenance)
and two production departments (Machining and Assembly). The company allocates Building
Maintenance cost on the basis of square footage and believes that Building Maintenance
provides more service than Human Resources. The square footage occupied by each
department follows.
Human Resources 3,500
Building Maintenance 8,700
Machining 9,900
Assembly 15,000

Over how many square feet would the Building Maintenance cost be allocated (i.e., spread)
with the direct method and the step-down method?

Direct Method Step-Down Method


A. 24,900 28,400
B. 24,900 37,100
C. 28,400 24,900
D. 37,100 24,900
AACSB: Analytic
E. Some other combination of figures not listed above. AICPA BB: Critical Thinking
AICPA FN: Measurement
Blooms: Apply
Difficulty: 3 Hard
Learning Objective: 17-01
Feedback A: Correct! Machining + Assembly = 9,900 + 15,000 = 24,900 sq. ft.; Machining + Assembly + HR = 9,900 + 15,000 + 3,500 =
28,400 sq. ft.
Feedback B: This amount combination is incorrect.
Feedback C: This amount combination is incorrect.
Feedback D: This amount combination is incorrect.
Feedback E: This answer is wrong, because there is a correct combination listed.

17-19
Chapter 17 - Allocation of Support Activity Costs and Joint Costs

Use the following information to answer Questions 40-42.

Marshall Welding Company has two service departments (Cafeteria and Human Resources)
and two production departments (Machining and Assembly). The number of employees in
each department follows.
Cafeteria 20
Human Resources 30
Machining 100
Assembly 150

40. Marshall Welding uses the step-down method of cost allocation and allocates cost on the
basis of employees. Human Resources cost amounts to $1,200,000, and the department
provides more service to the firm than Cafeteria. How much Human Resources cost would be
allocated to Machining? 
A. $0.
B. $428,572.
C. $444,444.
D. $480,000.
E. None of the answers is correct.

AACSB: Analytic
AICPA BB: Critical Thinking
AICPA FN: Measurement
Blooms: Apply
Difficulty: 3 Hard
Learning Objective: 17-01
Feedback A: This amount is incorrect.
Feedback B: This amount is incorrect.
Feedback C: Correct! Machining + Assembly + Cafeteria = 100 + 150 + 20 = 270; 100 ÷ 270 = 37%; $1,200,000 x 37% = $444,444.
Feedback D: This amount is incorrect.
Feedback E: This answer is wrong, because there is a correct amount listed.

 
41. Marshall Welding uses the step-down method of cost allocation and allocates cost on the
basis of employees. Human Resources cost amounts to $1,200,000, and the department
provides more service to the firm than Cafeteria. How much Human Resources cost would be
allocated to Assembly? 
A. $0.
B. $480,572.
C. $444,444.
D. $666,666.
E. None of the answers is correct.
 
AACSB: Analytic
AICPA BB: Critical Thinking
AICPA FN: Measurement
Blooms: Apply
Difficulty: 3 Hard
Learning Objective: 17-01
Feedback A: This amount is incorrect.
Feedback B: This amount is incorrect.
Feedback C: This amount is incorrect.
Feedback D: Correct! 150 ÷ 270 = 55.5%; $1,200,000 x 55.5% = $666,666
Feedback E: This answer is wrong, because there is a correct amount listed.

17-20
Chapter 17 - Allocation of Support Activity Costs and Joint Costs

 42. Marshall Welding uses the step-down method of cost allocation and allocates cost on the
basis of employees. Human Resources cost amounts to $1,200,000, and the department
provides more service to the firm than Cafeteria. How much Human Resources cost would be
allocated to Cafeteria? 
A. $88,888.
B. $28,572.
C. $44,444.
D. $0.
E. None of the answers is correct.
 
AACSB: Analytic
AICPA BB: Critical Thinking
AICPA FN: Measurement
Blooms: Apply
Difficulty: 3 Hard
Learning Objective: 17-01
Feedback A: Correct! 20 ÷ 270 = 7.4%; $1,200,000 x 7.4% = $88,888.
Feedback B: This amount is incorrect.
Feedback C: This amount is incorrect.
Feedback D: This amount is incorrect.
Feedback E: This answer is wrong, because there is a correct amount listed.

17-21
Chapter 17 - Allocation of Support Activity Costs and Joint Costs

43. Stormy Corporation has two service departments (S1 and S2) and two production
departments (P1 and P2), and uses the step-down method of cost allocation. Management has
determined that S1 provides more service to the firm than S2, and has decided that the number
of employees is the best allocation base to use for S1. The following data are available:

Department Number of Employees


S1 10
S2 20
P1 50
P2 70

Which of the following statements is (are) true if S1 and S2 have respective operating costs of
$280,000 and $350,000? 
A. S2 should allocate a portion of its $350,000 cost to S1.
B. S1's cost should be allocated (i.e., spread) over 140 employees.
C. S1's cost should be allocated (i.e., spread) over 150 employees.
D. S2 should allocate a total of $390,000 to P1 and P2.
E. Both S1's cost should be allocated (i.e., spread) over 140 employees and S2 should allocate
a total of $390,000 to P1 and P2.

 
AACSB: Analytic
AICPA BB: Critical Thinking
AICPA FN: Measurement
Blooms: Apply
Difficulty: 3 Hard
Learning Objective: 17-01
Feedback A: This statement is incorrect.
Feedback B: While this statement is true, there is a better answer choice listed.
Feedback C: This statement is incorrect.
Feedback D: While this statement is true, there is a better answer choice listed.
Feedback E: Correct! Both of these statements are true; S1 = S2 + P1 + P2 = 20 + 50 + 70 = 140; S1 allocation to P1 & P2 = $350,000 +
[($280,000 ÷ 140) x 20] = $390,000.

17-22
Chapter 17 - Allocation of Support Activity Costs and Joint Costs

Use the following information to answer Questions 44-46.

The Xtra Store has a Human Resources Department and a Janitorial Department that provide
service to three sales departments. The Human Resources Department cost is allocated on the
basis of employees, and the Janitorial Department cost is allocated on the basis of space. The
following information is available:

Human Janitorial Sales #1 Sales #2 Sales #3


Resources
Budgeted cost $45,000 $30,000
Space in Square Feet 4,000 1,000 20,000 30,000 50,000
Number of employees 5 10 15 45 30
 
44. Using the direct method, the amount of Janitorial Department cost allocated to Sales
Department no. 2 is: 
A. $8,571.
B. $8,654.
C. $9,000.
D. $10,350.
E. $14,210.
 
AACSB: Analytic
AICPA BB: Critical Thinking
AICPA FN: Measurement
Blooms: Apply
Difficulty: 3 Hard
Learning Objective: 17-01
Feedback A: This amount is incorrect.
Feedback B: This amount is incorrect.
Feedback C: Correct! 30,000 ÷ 100,000 = 30%; $30,000 x 30% = $9,000.
Feedback D: This amount is incorrect.
Feedback E: This amount is incorrect.

45. Using the step-down method and assuming that the Human Resources Department is
allocated first, the amount of Human Resources cost allocated to Sales Department no. 2 is: 
A. $12,000.
B. $12,857.
C. $13,500.
D. $15,000.
E. $22,500.
 
AACSB: Analytic
AICPA BB: Critical Thinking
AICPA FN: Measurement
Blooms: Apply
Difficulty: 3 Hard
Learning Objective: 17-01
Feedback A: This amount is incorrect.
Feedback B: This amount is incorrect.
Feedback C: Correct! 30,000 ÷ 100,000 = 30%; $45,000 x 30% = $13,500
Feedback D: This amount is incorrect.
Feedback E: This amount is incorrect.
 

17-23
Chapter 17 - Allocation of Support Activity Costs and Joint Costs

46. Using the step-down method and assuming the Human Resources Department is allocated
first, the amount of Janitorial cost allocated to Sales Department no. 2 is: 
A. $8,571.
B. $9,000.
C. $9,857.
D. $10,247.
E. $10,350.

 
AACSB: Analytic
AICPA BB: Critical Thinking
AICPA FN: Measurement
Blooms: Apply
Difficulty: 3 Hard
Learning Objective: 17-01
Feedback A: This amount is incorrect.
Feedback B: This amount is incorrect.
Feedback C: This amount is incorrect.
Feedback D: This amount is incorrect.
Feedback E: Correct! 34.5% x $30,000 = $10,350.

17-24
Chapter 17 - Allocation of Support Activity Costs and Joint Costs

Use the following information to answer Questions 47 & 48.

The Dahle Manufacturing Company has two production departments (Assembly and
Finishing) and two service departments (Human Resources and Janitorial). The projected
usage of the two service departments is as follows:

Use of Use of
Human Resources Janitorial
Human resource --- 5%
Janitorial 10% ---
Assembly 60% 40%
Finishing 30% 55%

The budgeted costs in the service departments are: Human Resources, $90,000 and Janitorial,
$50,000.

47. Using the direct method, the amount of Janitorial Department cost allocated to the
Finishing Department is: 
A. $21,053.
B. $24,843.
C. $25,000.
D. $28,947.
E. $34,157.

 
AACSB: Analytic
AICPA BB: Critical Thinking
AICPA FN: Measurement
Blooms: Apply
Difficulty: 3 Hard
Learning Objective: 17-01
Feedback A: This amount is incorrect.
Feedback B: This amount is incorrect.
Feedback C: This amount is incorrect.
Feedback D: Correct! 55 ÷ 95 = 57.89%; 57.895 % x $50,000 = $28,947
Feedback E: This amount is incorrect.

17-25
Chapter 17 - Allocation of Support Activity Costs and Joint Costs

48. Using the step-down method and assuming the Human Resources Department is allocated
first, the amount of Human Resources cost allocated to the Assembly Department is: 
A. $21,053.
B. $28,947.
C. $54,000.
D. $60,000.
E. $78,842.

 
AACSB: Analytic
AICPA BB: Critical Thinking
AICPA FN: Measurement
Blooms: Apply
Difficulty: 3 Hard
Learning Objective: 17-01
Feedback A: This amount is incorrect.
Feedback B: This amount is incorrect.
Feedback C: Correct! $90,000 x 60% = $54,000.
Feedback D: This amount is incorrect.
Feedback E: This amount is incorrect.
 

49. The process of allocating fixed and variable costs separately is called: 


A. the separate allocation procedure (SAP).
B. diverse allocation.
C. reciprocal-cost allocation.
D. common-cost allocation.
E. dual-cost allocation.

 
AACSB: Reflective Thinking
AICPA BB: Critical Thinking
AICPA FN: Research
Blooms: Remember
Difficulty: 1 Easy
Learning Objective: 17-02
Feedback A: This term is incorrect.
Feedback B: This term is incorrect.
Feedback C: This term is incorrect.
Feedback D: This term is incorrect.
Feedback E: Correct! This is the definition for dual-cost allocation.

17-26
Chapter 17 - Allocation of Support Activity Costs and Joint Costs

50. Under dual-cost allocation, fixed costs are allocated on the basis of a user department's: 
A. long-run usage of a service department's output.
B. short-run usage of a service department's output.
C. long-run usage and short-run usage of a service department's output.
D. neither long-run usage nor short-run usage of a service department's output.
E. either long-run usage or short-run usage of a service department's output.

 
AACSB: Reflective Thinking
AICPA BB: Critical Thinking
AICPA FN: Research
Blooms: Remember
Difficulty: 1 Easy
Learning Objective: 17-02
Feedback A: Correct! Under dual-cost allocation, fixed costs are allocated on the basis of a user department's long-run usage of a service
department's output.
Feedback B: This statement is incorrect.
Feedback C: This statement is incorrect.
Feedback D: This statement is incorrect.
Feedback E: This statement is incorrect.

51. Consider the following statements about dual-cost allocation:


I. Dual-cost allocation prevents a change in the short-run activity of one department from
affecting the cost allocated to another department.
II. Dual-cost allocations create an incentive for user department managers to understate their
expected long-run service needs.
III. Dual-cost allocations are generally preferred over lump-sum allocations, or those that
combine variable and fixed costs together.

Which of the above statements is (are) true? 


A. I only.
B. III only.
C. I and II.
D. II and III.
E. I, II, and III.

 
AACSB: Reflective Thinking
AICPA BB: Critical Thinking
AICPA FN: Research
Blooms: Remember
Difficulty: 1 Easy
Learning Objective: 17-02
Feedback A: While this statement is true, there is a better answer choice listed.
Feedback B: While this statement is true, there is a better answer choice listed.
Feedback C: While these statements are true, there is a better answer choice listed.
Feedback D: While these statements are true, there is a better answer choice listed.
Feedback E: Correct! All of these statements are true.

17-27
Chapter 17 - Allocation of Support Activity Costs and Joint Costs

52. When allocating service department costs, companies should use: 


A. actual costs rather than budgeted costs, and separate rates for variable and fixed costs.
B. budgeted costs rather than actual costs, and separate rates for variable and fixed costs.
C. budgeted costs rather than actual costs, and a rate that combines variable and fixed costs.
D. actual costs rather than budgeted costs, and a rate that combines variable and fixed costs.
E. a rate that is based on matrix theory.

 
AACSB: Reflective Thinking
AICPA BB: Critical Thinking
AICPA FN: Research
Blooms: Understand
Difficulty: 2 Medium
Learning Objective: 17-02
Feedback A: This statement is incorrect.
Feedback B: Correct! When allocating service department costs, companies should use budgeted costs rather than actual costs, and separate
rates for variable and fixed costs.
Feedback C: This statement is incorrect.
Feedback D: This statement is incorrect.
Feedback E: This statement is incorrect. 

53. Grassley Corporation allocates administrative costs on the basis of staff hours. Short-run
monthly usage and anticipated long-run monthly usage of staff hours for Operating
Departments 1 and 2 follow.

Department 1 Department 2 Total


Short-run usage (hours) 40,000 60,000 100,000
Long-run usage (hours) 45,000 55,000 100,000

If Grassley uses dual-cost accounting procedures and variable administrative costs total
$200,000, the amount of variable administrative cost to allocate to Department 1 would be: 
A. $80,000.
B. $85,000.
C. $90,000.
D. $100,000.
E. None of the answers is correct.

 
AACSB: Reflective Thinking
AICPA BB: Critical Thinking
AICPA FN: Research
Blooms: Apply
Difficulty: 3 Hard
Learning Objective: 17-02
Feedback A: Correct! (40,000 ÷ 100,000) x $200,000 = $80,000
Feedback B: This amount is incorrect.
Feedback C: This amount is incorrect.
Feedback D: This amount is incorrect.
Feedback E: This answer is wrong, because there is a correct amount listed.

17-28
Chapter 17 - Allocation of Support Activity Costs and Joint Costs

54. Soprano Corporation allocates administrative costs on the basis of staff hours. Short-run
monthly usage and anticipated long-run monthly usage of staff hours for Operating
Departments 1 and 2 follow.

Department 1 Department 2 Total


Short-run usage (hours) 80,000 120,000 200,000
Long-run usage (hours) 90,000 110,000 200,000

If Soprano uses dual-cost accounting procedures and fixed administrative costs total
$1,000,000, the amount of long-run cost to allocate to Department 1 would be: 
A. $400,000.
B. $450,000.
C. $500,000.
D. $850,000.
E. None of the answers is correct.

 
AACSB: Reflective Thinking
AICPA BB: Critical Thinking
AICPA FN: Research
Blooms: Apply
Difficulty: 3 Hard
Learning Objective: 17-02
Feedback A: This amount is incorrect.
Feedback B: Correct! (90,000 ÷ 200,000) x $1,000,000 = $450,000.
Feedback C: This amount is incorrect.
Feedback D: This amount is incorrect.
Feedback E: This answer is wrong, because there is a correct amount listed.

17-29
Chapter 17 - Allocation of Support Activity Costs and Joint Costs

55. Nichols Corporation allocates administrative costs on the basis of staff hours. Short-run
monthly usage and anticipated long-run monthly usage of staff hours for Operating
Departments 1 and 2 follow.

Department 1 Department 2 Total


Short-run usage (hours) 45,000 55,000 100,000
Long-run usage (hours) 48,000 52,000 100,000

Variable and fixed administrative costs total $180,000 and $400,000, respectively. If Nichols
uses dual-cost accounting procedures, the total amount of administrative cost to allocate to
Department 2 would be: 
A. $301,600.
B. $307,000.
C. $313,600.
D. $319,000.
E. None of the answers is correct.

AACSB: Analytic
AICPA BB: Critical Thinking
AICPA FN: Measurement
Blooms: Apply
Difficulty: 3 Hard
Learning Objective: 17-02
Feedback A: This amount is incorrect.
Feedback B: Correct! (55% x $180,000) + (52% x $400,000) = $99,000 + $208,000 = $307,000.
Feedback C: This amount is incorrect.
Feedback D: This amount is incorrect.
Feedback E: This answer is wrong, because there is a correct amount listed.
 

56. A company that uses activity-based costing would likely allocate costs from: 
A. service departments to production departments.
B. service departments to products and services.
C. service departments to production departments and then to products and services.
D. activity-cost pools to production departments.
E. activity-cost pools to products and services.

AACSB: Reflective Thinking


AICPA BB: Critical Thinking
AICPA FN: Research
Blooms: Remember
Difficulty: 1 Easy
Learning Objective: 17-03
Feedback A: This statement is incorrect.
Feedback B: This statement is incorrect.
Feedback C: This statement is incorrect.
Feedback D: This statement is incorrect.
Feedback E: Correct! A company that uses activity-based costing would likely allocate costs from activity-cost pools to products and
services.

17-30
Chapter 17 - Allocation of Support Activity Costs and Joint Costs

57. The point in a joint production process where each individual product becomes separately
identifiable is commonly called the: 
A. decision point.
B. separation point.
C. individual product point.
D. split-off point.
E. joint product point.

 
AACSB: Reflective Thinking
AICPA BB: Critical Thinking
AICPA FN: Research
Blooms: Remember
Difficulty: 1 Easy
Learning Objective: 17-04 
Feedback A: This term is incorrect.
Feedback B: This term is incorrect.
Feedback C: This term is incorrect.
Feedback D: Correct! This is the definition of a split-off point.
Feedback E: This term is incorrect.

58. The joint-cost allocation method that recognizes the revenues at split-off but does not
consider any further processing costs is the: 
A. relative-sales-value method.
B. net-realizable-value method.
C. physical-units method.
D. reciprocal-accounting method.
E. gross margin at split-off method.

 
AACSB: Reflective Thinking
AICPA BB: Critical Thinking
AICPA FN: Research
Blooms: Remember
Difficulty: 1 Easy
Learning Objective: 17-04 
Feedback A: Correct! This is the definition of a relative-sales-value method.
Feedback B: This term is incorrect.
Feedback C: This term is incorrect.
Feedback D: This term is incorrect.
Feedback E: This term is incorrect.

17-31
Chapter 17 - Allocation of Support Activity Costs and Joint Costs

59. Which of the following choices correctly denotes the data needed to allocate joint costs
under the relative-sales-value method?

Sales Value of Separable Sales Value of Product


Product at Split-Off Cost After Processing Beyond
Split-Off
A. Yes Yes No
B. Yes Yes Yes
C. Yes No No
D. No Yes Yes
E. No No Yes
AACSB:
Reflective Thinking
AICPA BB: Critical Thinking
AICPA FN: Research
Blooms: Understand
Difficulty: 2 Medium
Learning Objective: 17-04 
Feedback A: This answer choice is incorrect.
Feedback B: This answer choice is incorrect.
Feedback C: Correct! Data needed to allocate joint costs under the relative-sales-value method is sales value of product at split-off.
Feedback D: This answer choice is incorrect.
Feedback E: This answer choice is incorrect.

60. Which of the following methods should be selected if a company terminates all processing
at the split-off point and desires to use a cost-allocation approach that considers the "revenue-
producing ability" of each product? 
A. Gross margin at split-off method.
B. Reciprocal-accounting method.
C. Relative-sales-value method.
D. Physical-units method.
E. Net-realizable-value method.

 
AACSB: Reflective Thinking
AICPA BB: Critical Thinking
AICPA FN: Research
Blooms: Understand
Difficulty: 2 Medium
Learning Objective: 17-04 
Feedback A: This method is incorrect.
Feedback B: This method is incorrect.
Feedback C: Correct! This describes the relative-sales-value method.
Feedback D: This method is incorrect.
Feedback E: This method is incorrect.

17-32
Chapter 17 - Allocation of Support Activity Costs and Joint Costs

61. When allocating joint costs, Feinberg calculates the final sales value of the various
products manufactured and subtracts appropriate separable costs. The company is using the: 
A. gross margin at split-off method.
B. reciprocal-accounting method.
C. relative-sales-value method.
D. physical-units method.
E. net-realizable-value method.

 
AACSB: Reflective Thinking
AICPA BB: Critical Thinking
AICPA FN: Research
Blooms: Understand
Difficulty: 2 Medium
Learning Objective: 17-04
Feedback A: This is not the correct method.
Feedback B: This is not the correct method.
Feedback C: This is not the correct method.
Feedback D: This is not the correct method.
Feedback E: Correct! Feinberg is using the net-realizable-value method.
 

62. Hancock Machining manufactures A, B, and C, all of which are joint products, and D,
which is classified as a by-product. If joint manufacturing costs amount to $450,000 and the
company is using a popular accounting method, the firm will: 
A. allocate $450,000 among A, B, and C.
B. allocate $450,000 among A, B, C, and D.
C. increase $450,000 by the net realizable value of D and then allocate the total among A, B,
and C.
D. decrease $450,000 by the net realizable value of D and then allocate the total among A, B,
and C.
E. decrease $450,000 by the net realizable value of D and then allocate the total among A, B,
C, and D.

 
AACSB: Analytic
AICPA BB: Critical Thinking
AICPA FN: Measurement
Blooms: Apply
Difficulty: 3 Hard
Learning Objective: 17-04 
Feedback A: This statement is incorrect.
Feedback B: This statement is incorrect.
Feedback C: This statement is incorrect.
Feedback D: Correct! Hancock will decrease $450,000 by the net realizable value of D and then allocate the total among A, B, and C.
Feedback E: This statement is incorrect.

17-33
Chapter 17 - Allocation of Support Activity Costs and Joint Costs

63. Hsu Company manufactures two products (A and B) from a joint process that cost
$200,000 for the year just ended. Each product may be sold at the split-off point or processed
further. Additional processing requires no special facilities, and production costs of further
processing are entirely variable and traceable to the products involved. Further information
follows.

If Processed Further
Product Pounds Produced Per-Pound Sales Sales Value Separable
Price Cost
A 20,000 $12 $350,000 $90,000
B 30,000 8 300,000 60,000

If the joint costs are allocated based on the physical-units method, the amount of joint cost
assigned to product A would be: 
A. $80,000.
B. $100,000.
C. $104,000.
D. $120,000.
E. None of the answers is correct.

 
AACSB: Analytic
AICPA BB: Critical Thinking
AICPA FN: Measurement
Blooms: Apply
Difficulty: 3 Hard
Learning Objective: 17-04 
Feedback A: Correct! [20,000 ÷ (20,000 + 30,000)] x $200,000 = $80,000.
Feedback B: This amount is incorrect.
Feedback C: This amount is incorrect.
Feedback D: This amount is incorrect.
Feedback E: This answer is wrong, because there is a correct amount listed.

17-34
Chapter 17 - Allocation of Support Activity Costs and Joint Costs

64. Ramos Corporation uses the physical-units method to allocate costs among its three joint
products: X, Y, and Z. The following data are available for the period just ended:
Joint processing cost: $800,000
Total production: 150,000 pounds
Share of joint cost allocated to X: $160,000
Share of joint cost allocated to Y: $400,000

Which of the following statements is true? 


A. The company would have relied on the sales value of each product when allocating joint
costs to X, Y, and Z.
B. Ramos produced 30,000 pounds of Z during the period.
C. Ramos produced 45,000 pounds of Z during the period.
D. Ramos produced 105,000 pounds of Z during the period.
E. Based on the data presented, it is not possible to determine Ramos’ production of Z during
the period.

 
AACSB: Analytic
AICPA BB: Critical Thinking
AICPA FN: Measurement
Blooms: Apply
Difficulty: 3 Hard
Learning Objective: 17-04 
Feedback A: This statement is incorrect.
Feedback B: This amount is incorrect.
Feedback C: Correct! $800,000 - $160,000 - $400,000 = $240,000; $240 ÷ $800 = 0.3; 150,000 x 0.3 = 45,000 lbs. of Z.
Feedback D: This amount is incorrect.
Feedback E: This answer is wrong, because there is a correct amount listed.

17-35
Chapter 17 - Allocation of Support Activity Costs and Joint Costs

Use the following information to answer Questions 65 & 66.

Kingston Specialty Corporation manufactures joint products P and Q. During a recent period,
joint costs amounted to $80,000 in the production of 20,000 gallons of P and 60,000 gallons
of Q. Kingston can sell P and Q at split-off for $2.20 per gallon and $2.60 per gallon,
respectively. Alternatively, both products can be processed beyond the split-off point, as
follows:

P Q
Separable processing costs $15,000 $35,000
Sales price (per gallon) if processed beyond split-off $3 $4

65. The joint cost allocated to Q under the relative-sales-value method would be: 
A. $40,000.
B. $62,400.
C. $64,000.
D. $65,600.
E. None of the answers is correct.

 
AACSB: Analytic
AICPA BB: Critical Thinking
AICPA FN: Measurement
Blooms: Apply
Difficulty: 3 Hard
Learning Objective: 17-04
Feedback A: This amount is incorrect.
Feedback B: Correct! ($2.20 x 20,000) + ($2.60 x 60,000) = 44,000 + 156,000 = 200,000; (156,000 ÷ 200,000) x $80,000 = $62,400.
Feedback C: This amount is incorrect.
Feedback D: This amount is incorrect.
Feedback E: This answer is wrong, because there is a correct amount listed.

66. The joint cost allocated to P under the relative-sales-value method would be: 
A. $17,600.
B. $16,400.
C. $24,000.
D. $25,600.
E. None of the answers is correct.

AACSB: Analytic
AICPA BB: Critical Thinking
AICPA FN: Measurement
Blooms: Apply
Difficulty: 3 Hard
Learning Objective: 17-04 
Feedback A: ($2.20 x 20,000) ÷ [($2.20 x 20,000) + ($2.60 x 60,000)] = 22%; 22% x $80,000 = $17,600.
Feedback B: This amount is incorrect.
Feedback C: This amount is incorrect.
Feedback D: This amount is incorrect.
Feedback E: This answer is wrong, because there is a correct amount listed.

17-36
Chapter 17 - Allocation of Support Activity Costs and Joint Costs

Use this information to answer questions 67-68.

Corey Corporation manufactures joint products W and X. During a recent period, joint costs
amounted to $300,000 in the production of 20,000 gallons of W and 60,000 gallons of X.
Both products will be processed beyond the split-off point, giving rise to the following data:

W X
Separable processing costs $40,000 $160,000
Sales price (per gallon) if processed beyond split-off $14 $12

67. The joint cost allocated to W under the net-realizable-value method would be: 
A. $75,000.
B. $80,000.
C. $84,000.
D. $90,000.
E. None of the answers is correct.

 
AACSB: Analytic
AICPA BB: Critical Thinking
AICPA FN: Measurement
Blooms: Apply
Difficulty: 3 Hard
Learning Objective: 17-04 
Feedback A: This amount is incorrect.
Feedback B: This amount is incorrect.
Feedback C: This amount is incorrect.
Feedback D: Correct! [($14 x 20,000) - $40,000] = $240,000; [($12 x 60,000) - $160,000] = $560,000; $240,000 ÷ ($240,000 + $560,000)
= 30%; 30% x $300,000 = $90,000.
Feedback E: This answer is wrong, because there is a correct amount listed.

68. The joint cost allocated to X under the net-realizable-value method would be: 
A. $210,000.
B. $180,000.
C. $184,000.
D. $190,000.
E. None of the answers is correct.

AACSB: Analytic
AICPA BB: Critical Thinking
AICPA FN: Measurement
Blooms: Apply
Difficulty: 3 Hard
Learning Objective: 17-04 
Feedback A: Correct! [($14 x 20,000) - $40,000] = $240,000; [($12 x 60,000) - $160,000] = $560,000; $560,000 ÷ ($240,000 +$ 560,000)
= 70%; 70% x $300,000 = $210,000.
Feedback B: This amount is incorrect.
Feedback C: This amount is incorrect.
Feedback D: This amount is incorrect.
Feedback E: This answer is wrong, because there is a correct amount listed.

17-37
Chapter 17 - Allocation of Support Activity Costs and Joint Costs

Use the following information to answer Questions 69 – 71.

Stoney Brook Company produces two products (X and Y) from a joint process. Each product
may be sold at the split-off point or processed further. Additional processing requires no
special facilities, and production costs of further processing are entirely variable and traceable
to the products involved. Joint manufacturing costs for the year were $60,000. Sales values
and costs were as follows:

If Processed Further
Product Units Made Sales Price at Split-Off Sales Value Separable Cost
X 9,000 $40,000 $78,000 $10,500
Y 6,000 80,000 90,000 7,500
 

69. If the joint production costs are allocated based on the physical-units method, the amount
of joint cost assigned to product X would be: 
A. $20,000.
B. $24,000.
C. $30,000.
D. $36,000.
E. $40,000.

 
AACSB: Analytic
AICPA BB: Critical Thinking
AICPA FN: Measurement
Blooms: Apply
Difficulty: 3 Hard
Learning Objective: 17-04 
Feedback A: This amount is incorrect.
Feedback B: This amount is incorrect.
Feedback C: This amount is incorrect.
Feedback D: Correct! (9,000 ÷ 15,000) x $60,000 = $36,000
Feedback E: This amount is incorrect.

17-38
Chapter 17 - Allocation of Support Activity Costs and Joint Costs

70. If the joint production costs are allocated based on the relative-sales-value method, the
amount of joint cost assigned to product X would be: 
A. $20,000.
B. $27,000.
C. $33,000.
D. $40,000.
E. None of the answers is correct.

 
AACSB: Analytic
AICPA BB: Critical Thinking
AICPA FN: Measurement
Blooms: Apply
Difficulty: 3 Hard
Learning Objective: 17-04
Feedback A: Correct! [$40,000 ÷ ($40,000 + $80,000)] x $60,000 = $20,000.
Feedback B: This amount is incorrect.
Feedback C: This amount is incorrect.
Feedback D: This amount is incorrect.
Feedback E: This answer is wrong, because there is a correct amount listed.

71. If the joint production costs are allocated based on the net-realizable-value method, the
amount of joint cost assigned to product Y would be: 
A. $20,000.
B. $27,000.
C. $33,000.
D. $40,000.
E. None of the answers is correct.

 
AACSB: Reflective Thinking
AICPA BB: Critical Thinking
AICPA FN: Research
Blooms: Apply
Difficulty: 3 Hard
Learning Objective: 17-04
Feedback A: This amount is incorrect.
Feedback B: This amount is incorrect.
Feedback C: Correct! $78,000 - $10,500 = $67,500; $90,000 - $7,500 = $82,500; $67,500 + $82,500 = $150,000; $82,500 ÷ $150,000 =
55%; 55% x 60,000 = $33,000.
Feedback D: This amount is incorrect.
Feedback E: This answer is wrong, because there is a correct amount listed.

17-39
Chapter 17 - Allocation of Support Activity Costs and Joint Costs

72. Which of the following statements about joint-cost allocation is false? 


A. Joint-cost allocation is useful in deciding whether to further process a product after split-
off.
B. Joint-cost allocation is useful in making a profit determination about individual joint
products.
C. Joint-cost allocation is helpful in inventory valuation.
D. Joint-cost allocation can be based on the number of units produced.
E. Joint-cost allocation can be accomplished by using several different methods that focus on
sales value and product "worth."

 
AACSB: Reflective Thinking
AICPA BB: Critical Thinking
AICPA FN: Research
Blooms: Understand
Difficulty: 2 Medium
Learning Objective: 17-05
Feedback A: Correct! Joint cost allocation is not useful in deciding whether to further process a product after split-off.
Feedback B: This statement is true.
Feedback C:This statement is true.
Feedback D: This statement is true.
Feedback E: This statement is true.

73. Consider the following statements about joint product cost allocation:


I. Joint product cost is allocated because it is necessary for inventory valuation.
II. Joint product cost is allocated because it is necessary for making economic decisions about
individual products (e.g., sell at split-off or process further).
III. Joint cost may be allocated to products by using several different methods.

Which of the above statements is (are) correct? 


A. I only.
B. III only.
C. I and II.
D. I and III.
E. I, II, and III.

 
AACSB: Reflective Thinking
AICPA BB: Critical Thinking
AICPA FN: Research
Blooms: Remember
Difficulty: 1 Easy
Learning Objective: 17-05
Feedback A: While this statement is correct, there is a better answer choice listed.
Feedback B: While this statement is correct, there is a better answer choice listed.
Feedback C: One of these statements is not correct.
Feedback D: Correct! Statements I and III are correct.
Feedback E: One of these statements is not correct.
 

17-40
Chapter 17 - Allocation of Support Activity Costs and Joint Costs

74. Downtown Hospital has two service departments (Patient Records and Accounting) and
two "production" departments (Internal Medicine and Surgery). Which of the following
allocations would likely take place under the reciprocal-services method of cost allocation? 
A. Allocation of Accounting cost to Patient Records.
B. Allocation of Patient Records cost to Internal Medicine.
C. Allocation of Surgery cost to Accounting.
D. Allocation of Internal Medicine cost to Surgery.
E. Both Allocation of Accounting cost to Patient Records and Allocation of Patient Records
cost to Internal Medicine.

 
AACSB: Reflective Thinking
AICPA BB: Critical Thinking
AICPA FN: Research
Blooms: Understand
Difficulty: 2 Medium
Learning Objective: 17-01
Feedback A: While this allocation would likely take place, there is a better answer choice listed.
Feedback B: While this allocation would likely take place, there is a better answer choice listed.
Feedback C: This allocation would not be likely to take place.
Feedback D: This allocation would not be likely to take place.
Feedback E: Correct! Both of these allocations would likely take place under a reciprocal-services method of allocation.

75. State Hospital has two service departments (Patient Records and Accounting) and two
"production" departments (Internal Medicine and Surgery). Which of the following
allocations would not take place under the reciprocal-services method of cost allocation? 
A. Allocation of Accounting cost to Patient Records.
B. Allocation of Patient Records cost to Internal Medicine.
C. Allocation of Surgery cost to Accounting.
D. Allocation of Internal Medicine cost to Surgery.
E. Both Allocation of Surgery cost to Accounting and Allocation of Internal Medicine cost to
Surgery.

 
AACSB: Reflective Thinking
AICPA BB: Critical Thinking
AICPA FN: Research
Blooms: Understand
Difficulty: 2 Medium
Learning Objective: 17-01
Feedback A: This allocation would likely take place.
Feedback B: This allocation would likely take place.
Feedback C: While this allocation would likely not take place, there is a better answer choice listed.
Feedback D: While this allocation would likely not take place, there is a better answer choice listed.
Feedback E: Correct! Both of these allocations would likely not take place under a reciprocal-services method of allocation.

 
 

17-41
Chapter 17 - Allocation of Support Activity Costs and Joint Costs

Essay Questions

76. Tempest Industries has two service departments (General Factory and Human Resources)
and two production departments (Machining and Assembly). The company uses the direct
method of service-department cost allocation, allocating General Factory cost on the basis of
square feet and Human Resources cost on the basis of employees. Budgeted allocation-base
and operating data for the four departments follow.

General Human
Factory Resources Machining Assembly
Square feet 7,000 3,000 90,000 30,000
Employees 50 30 120 180
Machine hours 200 --- 80,000 20,000
Labor hours 45,000 25,000 100,000 150,000

Additional information:
· Budgeted costs of General Factory and Human Resources respectively amount to
$1,560,000 and $950,000.
· The anticipated overhead costs incurred directly in the Machining and Assembly
Departments respectively total $3,650,000 and $2,340,000.
· The manufacturing overhead application bases used by Tempest’s production departments
are: Machining, machine hours; Assembly, labor hours.
· Company policy holds that a department's overhead application rate is based on a
department's own overhead plus an allocated share of service-department cost.

Required:
A. Allocate the company's service-department costs to the producing departments.
B. Compute the overhead application rates for Machining and Assembly. 

17-42
Chapter 17 - Allocation of Support Activity Costs and Joint Costs

Solution:

A. Machining Assembly
General Factory
Allocation base: 90,000 + 30,000 = 120,000 sq. ft.
Machining: (90,000 ÷ 120,000) x $1,560,000 $1,170,000
Assembly: (30,000 ÷ 120,000) x $1,560,000 $390,000
Human Resources:
Allocation base: 120 + 180 = 300 employees
Machining: (120 ÷ 300) x $950,000 380,000
Assembly: (180 ÷ 300) x $950,000 570,000
Total $1,550,000 $960,000

B. Share of service department cost $1,550,000 $960,000


Departmental overhead cost 3,650,000 2,340,000
Total cost $5,200,000 $3,300,000

Application rates:
Machining: $5,200,000  80,000 machine hours = $65 per MH
Assembly: $3,300,000  150,000 labor hours = $22 per LH

 
AACSB: Analytic
AICPA BB: Critical Thinking
AICPA FN: Measurement
Blooms: Apply
Difficulty: 3 Hard
Learning Objective: 17-01
 

17-43
Chapter 17 - Allocation of Support Activity Costs and Joint Costs

77. Midwest Alabama State College has two service departments, the Library and Computing
Services that assist the School of Business and the School of Health. Budgeted costs of the
Library and Computing Services are $800,000 and $1,800,000, respectively. Usage of the
service departments' output during the year is anticipated to be:

Provider of Service
User of Services Library Computing Services
Library --- 10%
Computing Services --- ---
School of Business 20% 60%
School of Health 80% 30%

17-44
Chapter 17 - Allocation of Support Activity Costs and Joint Costs

Required:
A. Use the direct method to allocate the costs of the Library and Computing Services to the
School of Business and the School of Health.
B. Repeat requirement "A" using the step-down method. Midwest allocates the cost of
Computing Services first. 

Solution:

A. School of Business School of Health


Fraction Amount Fraction Amount
Library $800,000 2/10 $160,000 8/10 $640,000
Computing Service 1,800,000 6/9 1,200,000 3/9 600,000
Total $2,600,000 $1,360,00 $1,240,000
0

B. Computing services ($1,800,000):


Library (10%) $180,000
School of Business (60%) 1,080,000
School of Health (30%) 540,000

17-45
Chapter 17 - Allocation of Support Activity Costs and Joint Costs

Library ($800,000 + $180,000 = $980,000):


School of Business (20%) $196,000
School of Health (80%) 784,000

 
School of Business: $1,080,000 + $196,000 = $1,276,000
School of Health: $540,000 + $784,000 = $1,324,000

 
AACSB: Analytic
AICPA BB: Critical Thinking
AICPA FN: Measurement
Blooms: Apply
Difficulty: 3 Hard
Learning Objective: 17-01
 

17-46
Chapter 17 - Allocation of Support Activity Costs and Joint Costs

78. Claremore Electronics, Inc. manufactures gauges for automobile dashboards. The


company has two production departments, Molding and Assembly. There are three service
departments: Human Resources, Maintenance, and Engineering. Usage of services by the
various departments follows.

Human
Resources Maintenanc Engineering
e
Human Resources --- --- ---
Maintenance 5% --- ---
Engineering 5% 10% ---
Molding 40% 40% 75%
Assembly 50% 50% 25%

17-47
Chapter 17 - Allocation of Support Activity Costs and Joint Costs

The budgeted costs in Claremore’s service departments are: Human Resources, $180,000;
Maintenance, $270,000; and Engineering, $200,000. The company rounds all calculations to
the nearest dollar.
Required:
A. Use the direct method to allocate Claremore’s service department costs to the production
departments.
B. Determine the proper departmental sequence to use in allocating the company's service
costs by the step-down method.
C. Ignoring your answer in part "B," assume that Human Resources costs are allocated first,
Maintenance costs second, and Engineering costs third. Use the step-down method to allocate
Claremore’s service department costs. 

Solution:

A.

Molding Assembly
Fraction Amount Fraction Amount
Human Resources $180,000 40/90 $80,000 50/90 $100,000
Maintenance 270,000 40/90 120,000 50/90 150,000
Engineering 200,000 75/100 150,000 25/100 50,000
Total $650,000 $350,000 $300,000

B. First: Human Resources (serves two other service


departments)
Second: Maintenance (serves one other service

17-48
Chapter 17 - Allocation of Support Activity Costs and Joint Costs

department)
Third: Engineering (serves no other service departments)

C. Human Resources ($180,000):


Maintenance (5%) $180,000
Engineering (5%) 1,080,000
Molding (40%) 540,000
Assembly (50%) 90,000

Maintenance ($270,000 + $9,000 = $279,000):


Engineering (10%) $27,900
Molding (40%) 111,600
Assembly (50%) 139,500

Engineering ($200,000 + $9,000 + $27,900 = $236,900):


Molding (75%) $177,675
Assembly (25%) 59,225

Molding: $72,000 + $111,600 + $177,675 = $361,275


Assembly: $90,000 + $139,500 + $59,225 = $288,725

AACSB: Analytic
AICPA BB: Critical Thinking
AICPA FN: Measurement
Blooms: Apply
Difficulty: 3 Hard
Learning Objective: 17-01
 

17-49
Chapter 17 - Allocation of Support Activity Costs and Joint Costs

79. Dodge City Corporation is developing departmental overhead rates based on direct labor
hours for its two production departments, Molding and Assembly. The Molding Department
worked 20,000 hours during the period just ended, and the Assembly Department worked
40,000 hours. The overhead costs incurred by Molding and Assembly were $151,250 and
$440,750, respectively.
Two service departments, Repair and Power, directly support the two production departments.
These service departments have costs of $90,000 and $250,000, respectively. The following
schedule reflects the use of Repair and Power's output by the various departments:

Repair Power Molding Assembly


Repair (repair hours) 500 500 4,000
Power (kilowatt hours) 120,000 420,000 60,000

17-50
Chapter 17 - Allocation of Support Activity Costs and Joint Costs

Required:
A. Allocate the company's service department costs to production departments by using the
direct method.
B. Calculate the overhead application rates of the production departments. Hint: Consider
both directly traceable and allocated overhead when deriving your answer.
C. Allocate the company's service department costs to production departments by using the
step-down method. Begin with the Power Department, and round calculations to the nearest
dollar. 

Solution:

A. Molding Assembly
Fraction Amount Fraction Amount
Repair $ 90,000 0.5/4.5 $ 10,000 4.0/4.5 $ 80,000
Power 250,000 4.2/4.8 218,750 0.6/4.8 31,250
$340,000 $228,750 $111,250

B. Molding Assembl
y
Allocated service department costs $228,750 $111,250
Overhead costs, traceable to production departments 151,250 440,750
Total overhead costs $380,000 $552,000

17-51
Chapter 17 - Allocation of Support Activity Costs and Joint Costs

Direct labor hours ÷ 20,000 ÷ 40,000


Overhead rate per hour $19.00 $13.80

C. Power ($250,000):
Repair (120/600) $50,000
Molding (420/600) 175,000
Assembly (60/600) 25,000

Repair ($90,000 + $50,000 = $140,000):


Molding (500/4,500) $15,556
Assembly (4,000 / 4,500) 124,444

Molding: $175,000 + $15,556 = $190,556


Assembly: $25,000 + $124,444 = $149,444
 
AACSB: Analytic
AICPA BB: Critical Thinking
AICPA FN: Measurement
Blooms: Apply
Difficulty: 3 Hard
Learning Objective: 17-01
 

17-52
Chapter 17 - Allocation of Support Activity Costs and Joint Costs

80. Allegiance Corporation has two service departments (S1 and S2) and two production
departments (P1 and P2). S1 and S2 both use the number of employees as an allocation base.
The following data are available:

Number of Budgeted Cost


Employees
S1 40 $172,000
S2 60 250,000
P1 300 660,000
P2 500 840,000

Required:

A. Assuming use of the direct method:


1. Over how many employees would S1's budgeted cost be allocated?
2. How much of S2's cost would be allocated to P1?
3. How much of P1's cost would be allocated to S1?

B. Assuming use of the step-down method:


1. How much of S1's cost would be allocated to S2? Allegiance allocates S1's costs prior to
allocating those of S2.
2. How much of S2's total cost would be allocated to P2?
3. How much of S2's total cost would be allocated to S1? 

Solution:

A. 1. 800 (300 + 500)


2. $93,750 [$250,000  (300/800)]
3. None, because production department costs are not allocated to service departments.

B. 1. S1's costs are allocated over 860 employees (60 + 300 + 500). Thus, $12,000 will be
allocated to S2 [$172,000  (60/860)].
2. S2's costs total $262,000 ($250,000 + $12,000), resulting in $163,750 being allocated to P2
[$262,000  (500/800).
3. None, because S1's costs are allocated prior to those of S2. Once a department is closed, no
costs are allocated back to it.

 
AACSB: Analytic
AICPA BB: Critical Thinking
AICPA FN: Measurement
Blooms: Apply
Difficulty: 3 Hard
Learning Objective: 17-01
 

17-53
Chapter 17 - Allocation of Support Activity Costs and Joint Costs

81. Consider the following independent cases that relate to service department cost
allocations:
Case A: Aaron Company has two service departments [Human Resources (H/R) and
Information Systems] and two production departments (Machining and Assembly). Human
Resource cost is allocated by using the direct method based on the number of personnel in
each department. For the period just ended, there were 189 employees in Machining, and
Machining received $90,000 of H/R's overhead of $200,000. How many employees are in the
Assembly Department?
Case B: Drew Montana, controller of Butte Enterprises, wants service department managers
to be aware that their use of other service departments costs the firm a substantial amount of
money. Would Drew prefer the direct method or the step-down method of cost allocation?
Why?
Case C: Laramie Company has four service departments (S1, S2, S3, and S4) and two
production departments (P1 and P2). The costs of S1 are allocated first, followed in order by
the costs of S2, S3, and S4. Laramie uses the step-down method, and the costs of S2 are
allocated based on the number of computer hours used. Computer hours logged during the
period were as follows: S1, 4,600; S2, 7,100; S3, 10,400; S4, 17,600; P1, 37,000; and P2,
48,600. Over how many hours would S2's cost be allocated?
Case D: A recently hired staff accountant noted that given the nature of the allocations, the
total cost allocated to production departments is typically less under the step-down method
than under the direct method. Do you agree with the accountant? Why?

Required:
Answer the questions that are raised in Cases A, B, C, and D. 

Solution:

Case A: Machining has been allocated 45% of H/R's cost ($90,000  $200,000) because it has
45% of the employees in the production departments. Since 189 represents 45% of the total,
there are 420 employees in production (189  0.45). Thus, Assembly has 231 personnel (420 -
189).
Case B: Drew Montana would prefer the step-down method because service department costs
are allocated to other service departments (although not all). Such a practice makes managers
aware that services are not cost-free to the organization.
Case C: 113,600 (10,400 + 17,600 + 37,000 + 48,600)
Case D: No. Under both approaches, all service department costs are allocated to production
departments. This process yields equal totals for each method.

 
AACSB: Reflective Thinking
AICPA BB: Critical Thinking
AICPA FN: Measurement
Blooms: Apply
Difficulty: 3 Hard
Learning Objective: 17-01
 

17-54
Chapter 17 - Allocation of Support Activity Costs and Joint Costs

82. Carlson, Inc. has centralized much of its specialized data processing operation, with the
Computer Department performing services for Departments A and B. Service hours
consumed during Quarter No. 1 and Quarter No. 2 follow.

A B
Quarter No. 1 60 60
Quarter No. 2 40 60

Computer Department operating costs were:

Variable (Per Hour) Fixed


Quarter No. 1 $50 $40,000
Quarter No. 2 45 38,000

Company policy currently requires that total variable and fixed costs be combined and
allocated as a lump-sum to users based on service hours.
Carlson has been financially healthy for a number of years but began to experience problems
toward the end of Quarter No. 1. In response to these problems, management issued a
directive to closely monitor costs and computer usage, effective with the start of Quarter No.
2.

Required:
A. Compute Quarter No. 1's total computer cost and determine the allocation to Department A
and Department B.
B. How much cost would be allocated to Departments A and B during Quarter No. 2, and how
would the heads of these departments likely react to the allocations in light of management's
directive?
C. Assume that at the beginning of quarter no. 2, the company switched to dual-cost
allocations, with variable costs allocated based on current usage and fixed costs allocated
based on long-run average utilization. An analysis of projected usage found that work for
Department A was expected to consume 55% of the Computer Department's time over the
forthcoming year. How much cost would be allocated to A and B in Quarter No. 2?
D. Given the use of dual allocations, how, if at all, would a short-term increase or decrease in
A's current usage affect the quarterly cost allocation that is charged to Department B? 

17-55
Chapter 17 - Allocation of Support Activity Costs and Joint Costs

Solution:

A.

Variable cost: (60 + 60) x $50 $6,000


Fixed cost 40,000
Total cost $46,000

Since each department consumed 60 hours of services, the cost would be split equally:
$46,000  2 = $23,000.
B.

Variable cost: (40 + 60) x $45 $4,500


Fixed cost 38,000
Total cost $42,500

Department A: (40  100)  $42,500 = $17,000


Department B: (60  100)  $42,500 = $25,500
The head of A would be pleased because the department's decreased usage resulted in a
$6,000 reduction in cost ($23,000 - $17,000). In contrast, the head of B would likely be
unhappy. Although unable to reduce usage, usage did remain constant—a situation that
produced a $2,500 increase in cost ($25,500 - $23,000) despite the fact that overall cost
declined.
C.
Variable:
40 hours x $45 $1,800
60 hours x $45 $2,700
Fixed:
$38,000 x 55% 20,900
$38,000 x 45% 17,100
Total $22,700 $19,800

D. There is no effect on B. The variable costs charged to Department A would increase or


decrease, with other allocations remaining the same.

 
AACSB: Analytic
AICPA BB: Critical Thinking
AICPA FN: Decision Making
Blooms: Apply
Difficulty: 3 Hard
Learning Objective: 17-02
 

17-56
Chapter 17 - Allocation of Support Activity Costs and Joint Costs

83. Many companies use the dual-rate method of cost allocation.

Required:

A. How does the dual-rate method work?


B. Is there any advantage of the dual-rate method over a method that uses a combined, lump-
sum single rate? Briefly explain. 

Solution:

A. The dual-rate method involves creating two overhead rates, one for variable costs and
another for fixed costs. The variable costs are normally allocated on the basis of short-run
usage of the service department's output; fixed costs are allocated on the basis of long-run
usage.
B. Yes. When a single rate is used, the cost allocated to a user department may be influenced
by the amount of service consumed by another department. For example, a user department's
service consumption could remain flat; yet the amount of cost allocated to that department
could increase or decrease over previous amounts based solely on actions of other users. Dual
rates eliminate this problem.

 
AACSB: Reflective Thinking
AICPA BB: Critical Thinking
AICPA FN: Measurement
Blooms: Understand
Difficulty: 2 Medium
Learning Objective: 17-02

17-57
Chapter 17 - Allocation of Support Activity Costs and Joint Costs

84. Bowden Outfitters manufactures a complete line of running shoes. The firm has three
manufacturing departments: Molding, Component, and Assembly. There are also two service
departments: Power and Maintenance.
The soles of the shoes are manufactured in the Molding Department. The tops of the shoes
are manufactured in the Component Department. The shoes are then completed in the
Assembly Department. Varying amounts of materials, time, and effort are required for each of
the shoe models. The Power Department and Maintenance Department provide services to the
three manufacturing departments.
Bowden has always used a plantwide overhead rate. Direct-labor hours are used to assign
overhead to products. The predetermined overhead rate is calculated by dividing the
company’s total estimated overhead by the total estimated direct-labor hours to be worked in
the three manufacturing departments.
Christine Macguire, director of cost management, has recommended that Bowden use
departmental overhead rates. The planned operating costs and expected levels of activity for
the coming year have been developed by Macguire and are presented by department in the
following schedules. (All numbers are in thousands.)
Service Departments
Power Maintenance
Departmental activity measures:
Maximum capacity 1,000 kilowatt-hours Adjustable
Estimated usage for the coming year 800 kilowatt-hours 125 hours
Departmental costs:
Materials and supplies $ 6,000 $2,500
Variable labor 1,400 2,250
Fixed overhead 13,000 1,250
Total service department costs $20,400 $6,000

Manufacturing Departments
Molding Component Assembly
Department activity measures:
Direct-labor hours 1,500 3,000 2,500
Machine hours 1,875 1,125 –0–
Departmental costs:
Direct material $12,400 $30,000 $ 1,250
Direct labor 3,500 20,000 12,000
Variable overhead 4,500 11,000 20,500
Fixed overhead 18,500 7,200 7,100
Total departmental costs $38,900 $68,200 $40,850

Use of service departments:


Maintenance:
Estimated usage in labor 90 25 10
hours for the coming year
Power (in kilowatt-hours):
Estimated usage for the 360 320 120
coming year
Maximum allotted capacity 500 350 150

17-58
Chapter 17 - Allocation of Support Activity Costs and Joint Costs

Required:
1. Calculate the plantwide overhead rate for Bowden Company for the coming year using the
same method as used in the past.
2. Christine Macguire has been asked to develop departmental overhead rates for comparison
with the plantwide rate. The following steps are to be followed in developing the
departmental rates.
a. The Maintenance Department costs should be allocated to the three manufacturing
departments using the direct method.
b. The Power Department costs should be allocated to the three manufacturing
departments using the dual method combined with the direct method. Fixed costs are to
be allocated according to maximum allotted capacity, and variable costs are to be
allocated according to planned usage for the coming year.
c. Calculate departmental overhead rates for the three manufacturing departments using
a machine-hour cost driver for the Molding Department and a direct-labor-hour cost
driver for the Component and Assembly departments.

17-59
Chapter 17 - Allocation of Support Activity Costs and Joint Costs

Solution:
1. Plantwide overhead rates:
Departments (numbers in thousands)
Molding Component Assembly Total
Manufacturing departments:
 Variable overhead $ 4,500 $11,000 $20,500 $35,000
 Fixed overhead  18,500   7,200   7,100  32,800
  Total manufacturing
   department overhead $23,000 $18,200 $27,600 $67,800
Service departments:
 Power  20,400
 Maintenance   6,000
  Total estimated overhead $94,200
Estimated direct-labor hours (DLH):
 Molding   1,500
 Component   3,000
 Assembly   2,500
  Total estimated
   direct-labor hours   7,000

Plantwide overhead rate = Estimated overhead/ Estimated DLH

= $94,200 ÷ 7,000

= $13.46 per direct-labor hour

2. Departmental overhead Departments (numbers in thousands)


rates:
Service Manufacturing
Power Maintenance Molding Component Assembly
Departmental overhead
 costs $20,400 $  6,000 $23,000 $18,200 $27,600
a. Allocation of maintenance costs
  (direct method) Proportions:
90/125, 25/125, 10/125   (6,000)   4,320    1,200     480

b. Allocation of power costs (dual,


direct method) Fixed costs   
($13,000): Proportions: 500/1000,
350/1000, 150/1000  (13,000)   6,500   4,550   1,950
   Variable costs ($6,400):
   Proportions: 360/800, 320/800,
    120/800   (7,400)   3,330   2,960   1,110
   Total allocated departmental
   overhead costs $     0 $     0 $37,150 $26,910 $31,140

c. Cost driver 1,875 MH 3,000 DLH 2,500 DLH

  Rate (departmental overhead


   ÷ units of cost driver) $19.81 per MH $8.97 per DLH $12.46 per DLH
AACSB: Analytic
AICPA BB: Critical Thinking
AICPA FN: Decision Making
Blooms: Apply
Difficulty: 3 Hard

17-60
Chapter 17 - Allocation of Support Activity Costs and Joint Costs

Learning Objective: 17-03

85. Suppose that one hog yields 250 pounds of ham, 200 pounds of chops, and 50 pounds of
miscellaneous items. The sales value of ham is $1.80 per pound; chops, $2.50 per pound; and
miscellaneous items, $1.00 per pound. The hog costs $670, and processing costs are $30.

Required:

A. Determine the proper allocation of joint costs to the three products by using the physical-
units method.
B. Repeat part "B" by using the relative-sales-value method. 

Solution: 

A. Weight at Relative Allocation of


the Split-off Proportion Joint Cost*
Point
Ham 250 pounds 25/50 $350
Chops 200 pounds 20/50 280
Miscellaneous items 50 pounds 5/50 70
500 pounds $700
* $670 + $30
B. Sales Value Relative Allocation of
at Split-off Proportion Joint Cost
Ham (250 x $1.80) $450 4.5/10 $315
Chops (200 x $2.50) 500 5.0/10 350
Miscellaneous items (50 x $1.00) 50 0.5/10 35
$1,000 $700

 
AACSB: Analytic
AICPA BB: Critical Thinking
AICPA FN: Measurement
Blooms: Apply
Difficulty: 3 Hard
Learning Objective: 17-04 

17-61
Chapter 17 - Allocation of Support Activity Costs and Joint Costs

86. Quatro Corporation manufactures two chemicals (Flextra and Hydro) in a joint process.
Data from a recent month follow.
Direct materials used: $360,000
Direct labor: $150,000
Manufacturing overhead: $690,000
Manufacturing output:
Flextra: 40,000 gallons
Hydro: 120,000 gallons
Flextra sells for $15 per gallon and Hydro sells for $20 per gallon.

Required:
A. Compute the total joint costs to be allocated to Flextra and Hydro.
B. Compute the joint costs that would be allocated to Flextra by using the physical-units
method.
C. Compute the joint costs that would be allocated to Hydro by using the relative-sales-value
method.
D. Assume that Hydro can be converted into a more refined product, Hydro-R, in a totally
separable process at an additional cost of $4 per gallon. If the refined product can be sold in
the marketplace for $26 per gallon, compute the net realizable value of Hydro-R. 

Solution:

A. $1,200,000 ($360,000 + $150,000 + $690,000)


B. Flextra constitutes 25% of the productive output [40,000  (40,000 + 120,000)] and would
therefore absorb $300,000 of joint cost ($1,200,000  25%).
C. The total sales value of the two products is $3,000,000: Flextra (40,000 gallons  $15 =
$600,000) + Hydro (120,000 gallons  $20 = $2,400,000). Since Hydro has 80% of the sales
value ($2,400,000  $3,000,000), the company will allocate $960,000 of joint cost
($1,200,000  80%).
D. Sales value (120,000 gallons  $26 = $3,120,000) - costs beyond split-off (120,000 gallons
 $4 = $480,000) = $2,640,000.

 
AACSB: Analytic
AICPA BB: Critical Thinking
AICPA FN: Measurement
Blooms: Apply
Difficulty: 3 Hard
Learning Objective: 17-04

17-62
Chapter 17 - Allocation of Support Activity Costs and Joint Costs

87. Lowrey Chemical manufactures two industrial chemicals in a joint process. In October,


$200,000 of direct materials were processed at a cost of $300,000, resulting in 16,000 pounds
of Pentex and 4,000 pounds of Glaxco. Pentex sells for $35 per pound and Glaxco sells for
$60 per pound. Management generally processes each of these chemicals further in separable
processes to manufacture more refined products. Pentex is processed separately at a cost of
$7.50 per pound, with the resulting product, Pentex-R, selling for $45 per pound. Glaxco is
processed separately at a cost of $10 per pound, and the resulting product, Glaxco-R, sells for
$100 per pound.

Required:
A. Compute the company's total joint production costs.
B. Assuming that total joint production costs amounted to $500,000, allocate these costs by
using: (1) The physical-units method; (2) The relative-sales-value method; (3) The net-
realizable-value method. 

Solution:
A. Joint production costs total $500,000 ($200,000 + $300,000).
B.
1. Weight at the Relative Allocation
Split-off Point Proportion of Joint
Cost
Pentex 16,000 16/20 $400,000
Glaxco 4,000 4/20 100,000
20,000 $500,000

2. Sales Relative Allocation


Value at Proportion of Joint
Split-off Cost
Pentex (16,000 x $560,000 56/80 $350,000
$35)
Glaxco (4,000 x $60) 240,000 24/80 150,000
$800,000 $500,000

3. Sales Value of Costs Past Net Relative Allocation


Final Product* Split-Off** Realizable Proportion of Joint
Value Cost
Pentex-R $720,000 $120,000 $600,000 60/96 $312,500
Glaxco-R 400,000 40,000 360,000 36/96 187,500
$1,120,000 $160,000 $960,000 $500,000
* Pentex-R: 16,000 x $45; Glaxco-R: 4,000 x $100
** Pentex-R: 16,000 x $7.50; Glaxco-R: 4,000 x $10AACSB: Analytic
AICPA BB: Critical Thinking
AICPA FN: Measurement
Blooms: Apply

17-63
Chapter 17 - Allocation of Support Activity Costs and Joint Costs

Difficulty: 3 Hard
Learning Objective: 17-04 

88. Clarion Company, a new firm, manufactures two products, J and K, in a common process.
The joint costs amount to $80,000 per batch of finished goods. Each batch results in 20,000
liters of output, of which 80% are J and 20% are K.
The two products are processed beyond the split-off point, with Clarion incurring the
following separable costs: J, $2 per liter; K, $5 per liter. After the additional processing, the
selling price of J is $12 per liter, and the selling price of K is $15 per liter.

Required:
A. Determine the proper allocation of joint costs if the company uses the net-realizable-value
method.
B. Assume that Clarion sold all of its production of K during the current accounting period.
Compute K's sales revenue, cost of goods sold, and gross margin.
C. Is the firm's cost-of-goods-sold figure influenced by the choice of a joint-cost allocation
method? Briefly explain. 

Solution:

A. Sales Costs Net Relative Allocation


Value of Past Realizable Proportion of Joint
Final Split-Off Value Cost
Product
J (16,000 x $12) $192,000 $32,000 $160,000 160/200 $64,000
K (4,000 x $15) 60,000 20,000 40,000 40/200 16,000
$200,000 $80,000
B.
Joint Costs $16,000
Costs beyond split- 20,000
off
Cost of goods sold $36,000

Sales Revenue $60,000


Cost of goods sold 36,000
Gross Margin $24,000

17-64
Chapter 17 - Allocation of Support Activity Costs and Joint Costs

C. Yes. Cost of goods sold is based on both separable costs and joint cost. The choice of an
allocation method will influence the amount of joint cost charged to the product.

 
AACSB: Analytic
AICPA BB: Critical Thinking
AICPA FN: Measurement
Blooms: Apply
Difficulty: 3 Hard
Learning Objective: 17-04
Learning Objective: 17-05
 

89. Palen Chemical Company manufactures X-111, X-112, and X-113 from a joint process.
The following information is available for the period just ended:

X-111 X-112 X-113 Total


Units produced 6,000 14,000 30,000 50,000
Joint cost allocation ? $18,400 ? $80,000
Sales value at split-off $104,000 ? ? $260,000

Required:
A. Does Palen allocate joint costs by using the physical-units method? Explain.
B. Assume that Palen does not use the physical-units method but instead allocates joint costs
by using the relative-sales-value method. Find the four unknowns in the preceding table. 

Solution:

A. No. X-112 comprises 28% of the total units produced (14,000  50,000); however, the
product was allocated 23% of the total joint cost ($18,400  $80,000). Apparently, then,
another method is being used.
B. X-111 has 40% of the sales value ($104,000  $260,000), resulting in 40% of the joint cost
($80,000  40%), or $32,000. This leaves $29,600 to be allocated to X-113 ($80,000 -
$32,000 - $18,400). The sales values follow by using the same percentages that are used in the
cost allocation ($18,400  $80,000 = 23%; $260,000  23% = $59,800) and ($29,600 
$80,000 = 37%; $260,000  37% = $96,200).

X-111 X-112 X-113 Total


Units produced 6,000 14,000 30,000 50,000
Joint cost allocation $32,000 $18,400 $29,600 $80,000
Sales value at split-off $104,000 $59,800 $96,200 $260,000

17-65
Chapter 17 - Allocation of Support Activity Costs and Joint Costs

AACSB: Analytic
AICPA BB: Critical Thinking
AICPA FN: Measurement
Blooms: Apply
Difficulty: 3 Hard
Learning Objective: 17-04 

90. Clandestine Corporation allocates joint costs by using the net-realizable-value method. In


the company's Texas plant, products D and E emerge from a joint process that costs $250,000.
E is then processed at a cost of $220,000 into products F and G. Data pertaining to D, F, and
G follow.

D F G
Costs beyond split-off $50,000 $27,000 $25,000
Selling price 40 38 50
Pounds produced 10,000 4,000 2,000

Required:
A. Allocate the $220,000 processing cost between products F and G.
B. From a profitability perspective, should product E be processed into products F and G?
Show your calculations.
C. Assume that the net realizable value associated with E is zero. How would you allocate the
joint cost of $250,000? 

Solution: 

A. Sales Value Costs Past Net Relative Allocation of


of Final Split-Off Realizable Proportion Joint Cost
Product Value
F (4,000 x $38) $152,000 $27,000 $125,000 125/200 $137,500
G (2,000 x $50) 100,000 25,000 75,000 75/200 82,500
$200,000 $220,000

17-66
Chapter 17 - Allocation of Support Activity Costs and Joint Costs

B. No, the company is losing $20,000: Net realizable value ($200,000) - joint costs
($220,000).
C. The $250,000 cost is a joint cost between D and E. Since product D has a positive net
realizable value of $350,000 [(10,000 pounds  $40) - $50,000] and E's is zero, all $250,000
would be charged to D.

 
AACSB: Analytic
AICPA BB: Critical Thinking
AICPA FN: Measurement
Blooms: Apply
Difficulty: 3 Hard
Learning Objective: 17-04 

91. Companies are free to use the direct, step-down, and reciprocal allocation methods when
dealing with service-department costs.

Required:
A. How does the direct method work? What is its chief limitation?
B. Is the step-down method an improvement over the direct method? Explain.
C. Which of the three methods is the most correct from a conceptual viewpoint? Why? 

Solution:

A. The direct method allocates joint costs solely to producing departments. This method does
not allocate costs to other service departments and is based on the erroneous assumption that
service departments do not service each other.
B. The step-down method is an improvement over the direct method, as it recognizes that
service departments service both producing departments and other service departments. Costs
are allocated accordingly. The step-down method is slightly more complex than the direct
method, requiring a determination of the proper order of departmental allocations.
C. The reciprocal method is the most correct approach from a conceptual viewpoint. This
method fully recognizes all services provided by service departments. The direct method
completely ignores the fact that service departments service each other. The step-down
method recognizes only some of these services, as once a department is closed out, no cost is
reallocated back to it.

 
AACSB: Reflective Thinking
AICPA BB: Critical Thinking
AICPA FN: Measurement
Blooms: Understand
Difficulty: 2 Medium
Learning Objective: 17-01
Learning Objective: 17-06
 

17-67
Chapter 17 - Allocation of Support Activity Costs and Joint Costs

92. Crystal Magic Company is developing departmental overhead rates based on direct-labor
hours for its two production departments in the production of light-catcher art pieces, Etching
and Assembly. The Etching Department employs 12 people and the Assembly Department
employs 48 people. Each person in these two departments works 2,000 hours per year. The
production-related overhead costs for the Etching Department are budgeted at $400,000, and
the Assembly Department costs are budgeted at $640,000. Two service departments,
Maintenance and Computing, directly support the two production departments. These service
departments have budgeted costs of $96,000 and $500,000, respectively. The production
departments’ overhead rates cannot be determined until the service departments’ costs are
allocated. The following schedule reflects the use of the Maintenance Department’s and
Computing Department’s output by the various departments.

Using Department
Service Department Maintenance Computing Etching Assembly
Maintenance (maintenance hours) 0 1,000 1,000 8,000
Computing (minutes) 240,000 0 840,000 120,000

Required:
(Use M for Maintenance and C for Computing in your equations.)
1. What is the equation for the total cost of the maintenance department when using the
reciprocal services method?
2. What is the equation for the total cost of the computing department when using the
reciprocal services method?
3. Solve each equation.
4. Using the reciprocal-services method to allocate service department costs, calculate the
overhead rates per direct-labor hour for the Etching Department and the Assembly
Department.

17-68
Chapter 17 - Allocation of Support Activity Costs and Joint Costs

Solution:
1. Reciprocal-services method: Equations for Maintenance: M = 96,000 + .2C
2. Reciprocal-services method: Equations for Computing: C = 500,000 + .1M
3. Solution of equations:
M = 96,000 + .2 (500,000 + .1M)
M = 96,000 + 100,000 + .02M
.98 M = 196,000
M = 200,000
C = 500,000 + .1 (200,000)
C = 520,000

4. Service Departments Production Departments


Maintenance Computing Etching Assembly
Traceable costs $ 96,000 $500,000
Allocation of Maintenance $ 
 Department costs (200,000) 20,000(.1) $ 20,000(.1) 160,000(.8)
Allocation of Computing
 Department costs 104,000(.2) (520,000)  364,000(.7)   52,000(.1)
Total service department costs allocated $384,000 $212,000
Overhead costs traceable to production departments  400,000  640,000
Total overhead cost $784,000 $852,000
Direct-labor hours (DLH)
 (12  2,000) 24,000
 (48  2,000) 96,000
Overhead rate per hour (total overhead ÷ DLH) $32.67 $8.88
Check on allocation procedure:
 Service department costs allocated to Etching $384,000
 Service department costs allocated to Assembly  212,000
 Total $596,000

AACSB: Analytic
AICPA BB: Critical Thinking
AICPA FN: Measurement
Blooms: Apply
Difficulty: 3 Hard
Learning Objective: 17-06

17-69

You might also like